which of the following facts should make you the most worried about the reliability of the results of the test in this case? there is no need to worry because all of the expected cell counts are above 5. the sample was only 300 black women and is not representative of all black women. two of the six observed counts are 5 or less. not all of the cells' contributions to the chi-squared statistic are greater than 1. two of the six expected counts are less than 5.
Therefore, the reliability of the results may be questioned due to the small sample size and the fact that the observed and expected counts in some cells are too small.
The fact that two of the six observed counts are 5 or less should make you the most worried about the reliability of the results of the test in this case. This is because when the observed counts in a cell are too small, it is difficult to draw reliable conclusions from them. In such cases, the chi-squared statistic may not accurately reflect the true relationship between the variables being studied. This is because the chi-squared statistic is based on the difference between the observed counts and the expected counts, and if the observed counts are too small, the chi-squared statistic may be biased and not accurately reflect the true relationship between the variables.
Moreover, the fact that two of the six expected counts are less than 5 also raises concerns about the reliability of the results. This is because when the expected counts in a cell are too small, it may indicate that the sample size is too small or that the sample is not representative of the population being studied. In this case, the sample size was only 300 black women, which may not be representative of all black women.
To know more about chi-squared statistic visit:
https://brainly.com/question/31036349
#SPJ11
Two of which shape can make a circle
Answer:
2 semi circles put together
Step-by-step explanation:
just cut the circle in half
can you make two triangles that are not congruent that have three pairs of congruent angles?
if two triangles have three pairs of congruent angles, then they must be congruent to each other by the Angle-Angle-Angle (AAA) congruence theorem, and thus cannot be non-congruent.
No, it is not possible to make two triangles that are not congruent and have three pairs of congruent angles. This is because if two angles of a triangle are congruent, then the third angle must also be congruent by the Angle Sum Theorem, which states that the sum of the angles in a triangle is always 180 degrees. If two triangles have three pairs of congruent angles, then all three angles in each triangle are congruent, meaning they have the same measure. However, this does not guarantee that the sides of the triangles are congruent. In order for two triangles to be congruent, they must have the same angle measures as well as the same side lengths. Therefore, if two triangles have three pairs of congruent angles, then they must be congruent to each other by the Angle-Angle-Angle (AAA) congruence theorem, and thus cannot be non-congruent.
To know more about congruent Triangle Visit:
https://brainly.com/question/29116501
#SPJ11
yesterday, 28 students took a test. the arithmetic mean of those 28 scores was 72 points. two students who were absent yesterday took the test this morning, and the arithmetic mean of all 30 test scores is 73 points. if the difference of the two scores from this morning is 22 points, what is the lower score from this morning?
Let's assume the sum of the scores of the 28 students who took the test yesterday is 28 * 72 = 2016.
We know that the sum of the scores of all 30 students is (28 * 72) + x + y, where x and y are the scores of the two students who took the test this morning.
We also know that the mean of all 30 scores is 73. So we can write an equation:
[(28 * 72) + x + y]/30 = 73
Multiplying both sides by 30, we get:
2016 + x + y = 2190
So, x + y = 174.
We are given that the difference of the two scores is 22, so we can write another equation:
y - x = 22
Solving these two equations simultaneously, we get y = 98 and x = 76.
Therefore, the lower score from this morning is 76.
To learn more refer below:
https://brainly.com/question/18495602
#SPJ11
In □PQRS side PQ∥ side RS. If m∠P = 108degree
and m∠R = 53degree
, then find m∠Q and m∠S.
In □PQRS side PQ∥ side RS. If m∠P = 108degree
and m∠R = 53degree
To Find :-m∠Q and m∠S.
Solution :-According to angle sum property
P∠Q=180−∠P
∠Q=180−108
\(\boxed{\sf{\angle Q = 72°}}\)
For angle S
∠S=180−∠R
∠S=180−53
\(\boxed{\sf {\angle S = 127°}}\)
\(\maltese\bold {lucky75} \maltese\)
Answer:
Step-by-step explanation:
In □PQRS side PQ∥ side RS
so m∠P and m∠S are interior angle pair which add to 180degree
given m∠P = 108degree
m∠S = 180 - 108 = 72degree
similarly m∠R and m∠Qare interior angle pair which add to 180degree
given m∠R = 53degree
m∠S = 180 - 53 = 127degree
Hello! I need help PLZ!! If anyone knows I would appreciate it! Thank you so much! :)
(4 screenshots included)
Answer:
its either b or d
Step-by-step explanation:
b or d is the answer
Suriland cannot both export wheat and keep bread plentiful and affordable in Suriland. Accordingly, Suriland's wheat farmers are required to sell their crop to the government, which pays them a dollar per bushel less than the price on the world market. Therefore, if the farmers could sell their wheat on the world market, they would make a dollar per bushel more, less any additional transportation and brokerage costs they would have to pay.
Which of the following, if true, most seriously weakens the argument?
(A) Suriland's wheat farmers have higher production costs than do farmers in many other wheat-producing countries.
(B) Sale of a substantial proportion of Suriland's wheat crop on the world market would probably depress the price of wheat.
(C) The transportation and brokerage costs that Suriland's farmers would face if they sold their wheat outside Suriland could amount to almost a dollar per bushel.
(D) Suriland is surrounded by countries that do not import any wheat.
(E) The price of a bushel of wheat on the world market occasionally drops below the average cost of producing a bushel of wheat in Suriland.
Answer:
the most accurate argument is (C) The transportation and brokerage costs that Suriland's farmers would face if they sold their wheat outside Suriland could amount to almost a dollar per bushel.
Step-by-step explanation:
The cost of transportation and other logistics is being covered by the government that is why the government pay less than dollar per bushel.
so the price of the wheat would have to decrease at home so that the cost of the transportation can be covered by the government who are buying directly from the farmers.
Simplify the radical. 2/√6 A)√12/6 B)√3/3 C)√6/3 D)√2
This is the square root of 6 over 3. The 3 is not inside the square root.
========================================================
Explanation:
sqrt = square root
We have sqrt(6) in the denominator. Your teacher wants you to rationalize the denominator, which means to take out the square root and instead have an integer here.
To achieve this, we multiply top and bottom of the fraction by sqrt(6) like so
\(\frac{2}{\sqrt{6}}\\\\\frac{2*\sqrt{6}}{\sqrt{6}*\sqrt{6}}\\\\\frac{2\sqrt{6}}{\sqrt{6*6}}\\\\\frac{2\sqrt{6}}{\sqrt{36}}\\\\\frac{2\sqrt{6}}{6}\\\\\frac{2\sqrt{6}}{2*3}\\\\\frac{\sqrt{6}}{3}\\\\\)
Therefore,
\(\frac{2}{\sqrt{6}}=\frac{\sqrt{6}}{3}\\\\\)
A bond that pays interest semiannually has a price of $958.56 and a semiannual coupon payment of $29.50. If the par value is $1,000, what is the current yield?
a. 5.90%
b. 3.08%
c. 5.85%
d. 2.95%
e. 6.16%
We need to find the current yield of a bond that pays interest semiannually, given that the bond's price is $958.56, the semiannual coupon payment is $29.50, and the par value is $1,000. Therefore, the answer is option (e) 6.16%.
The formula for current yield is given by (Annual Interest Payment / Current Market Price) * 100.
The bond that pays interest semiannually has a price of $958.56 and a semiannual coupon payment of $29.50.
If the par value is $1,000, what is the current yield?
Solution: Given, Price of the bond = $958.56
Semi-annual coupon payment = $29.50
The par value of the bond = $1000
The coupon rate per period is calculated as follows:
Coupon rate = Semi-annual coupon payment / Par value of the bond
Coupon rate = $29.50 / $1000Coupon rate = 0.0295
The current yield is calculated as follows:
Current yield = (Annual Interest Payment / Current Market Price) * 100
Since the bond pays interest semiannually, the annual interest payment is:
Annual Interest Payment = Semi-annual coupon payment * 2
Annual Interest Payment = $29.50 * 2
Annual Interest Payment = $59
The current market price of the bond is $958.56
Therefore, Current yield = (Annual Interest Payment / Current Market Price) * 100
Current yield = ($59 / $958.56) * 100
Current yield = 6.16%.
To learn more about current yield, refer:-
https://brainly.com/question/20164529
#SPJ11
as it may not be much, It's the best i can do, with such little wealth, take my earning, consider thy points as your spoils.
1^2 +68=69 skippa bap
Answer:
Thank you!
Step-by-step explanation:
Have a nice day/night lol
Answer:
1^2 +68=69
Step-by-step explanation:
by sqrt rooting 2 by 1 is 1
Calculate the simple interest earned on an investment of $7250 at a semiannual rate of 2.1% for 6 years.
Write your answer to the nearest cent.
Answer:
the simple interest=913.5
914 (Nearest cent)
Step-by-step explanation:
\(i = \frac{prt}{100} \\ i = \frac{7250 \times 2.1 \times 6}{100} \\ i = \frac{91,350}{100} \\ i = 913.5\)
\(I = 914 \\ to\: the\: nearest \:cent\)
The sum of the perimeters of 2 different squares is 68 feet, and the difference between their perimeters is 12 feet. What is the side length of
each square?
9514 1404 393
Answer:
10 ft7 ftStep-by-step explanation:
The side lengths are 1/4 of the perimeter, so we can say ...
the sum of side lengths is 68 ft/4 = 17 ft
the difference of side lengths is 12 ft/4 = 3 ft
Using x to represent the side length of the smaller square, we have ...
x + (x +3) = 17 . . . . . . the larger square has side length x+3
2x = 14 . . . . . . . . . subtract 3
x = 7 . . . . . . . . divide by 2
The side lengths of the squares are 7 feet and 10 feet.
A van travels 180 miles on 6 gallons of gas. How many gallons will it need to travel 750 miles?
The van would need approximately 25 gallons of gas to travel 750 miles.
What is a ratio?
A ratio is a quantitative relationship or comparison between two or more quantities. It represents the relative sizes or amounts of different things. Ratios are expressed using two numbers separated by a colon (:) or by using a fraction.
To solve this problem, we can set up a proportion using the information provided. Let's denote the number of gallons needed to travel 750 miles as "x." The proportion can be set up as follows:
180 miles / 6 gallons = 750 miles / x gallons
To solve for x, we can cross-multiply and then divide to isolate x:
180 * x = 6 * 750
180x = 4500
x = 4500 / 180
x ≈ 25
Therefore, the van would need approximately 25 gallons of gas to travel 750 miles.
To learn more about ratio and proportion follow the link:
https://brainly.com/question/12024093
#SPJ4
The Yellow Cab Company charges just $1.25 per mile, but it costs $6.00 to get in the cab. Express Cab charges no fee to get in the cab, but $2.00 per mile for the ride. For what number of miles is the cost the same?
Answer: 8 miles
Step-by-step explanation:
1.25m+6=2m
--1.25m. -1.25m
6=.75
6/.75m=8 mile
s
What is the slope of the line that passes through the points (6, 7) and (4, 1)?
Answer:
m = 3
Step-by-step explanation:
\(m=\frac{1-7}{4-6}=\frac{-6}{-2}=\boxed{3}\)
Hope this helps.
Answer:
3
Step-by-step explanation:
y¹-y²
------
x¹-x²
substitute then solve.
7-1 6
------ --> ----
6-4 2
6/2=3 so slope is 3.
How is this one done?
Given: DF and DE are midsegments of ∆ABC Prove: DE= 1/2 AC (PLEASE HELP)
Answer:
The correct answer is D because we proved that the triangles ΔDBE and ΔADF are congruent so that means that DE = segment A F because of CPCTC.
Answer:
The correct answer is D because we proved that the triangles ΔDBE and ΔADF are congruent so that means that DE = segment A F because of CPCTC.
Step-by-step explanation:
in a survey of patients in a local hospital, 62.42% of the respondents indicated that the health care providers needed to spend more time with each patient. who makes up the population? a. hospital patients b. all survey respondents c. all patients in a local hospital d. cannot be determined from the information given.
The option (c) all patients in a local hospital makes up the population
Surveys are a commonly used method for collecting data in various fields, including healthcare. They can provide valuable insights into patients' experiences, opinions, and preferences, which can help healthcare providers improve their services.
In this case, the survey was conducted among patients in a local hospital, and the results showed that 62.42% of the respondents believed that healthcare providers needed to spend more time with each patient.
The answer is not immediately obvious, but we can make some inferences based on the information given. However, we cannot generalize the results to all patients in the area or the entire population.
Third, surveys cannot account for all the factors that may influence the results, such as demographic characteristics, health status, or cultural background.
Therefore, we can conclude that the population under study is "patients in a local hospital" who participated in the survey.
To know more about survey here.
https://brainly.com/question/13532910
#SPJ4
The Venn diagram below is used for showing odd numbers and prime numbers.
Place the numbers 1, 2, 3, 4 and 5 in the Venn diagram.
E
Odd
numbers
Prime
numbers
The Venn Diagram for showing odd numbers and prime numbers would be :
Odd numbers :
1Prime number :
2Both prime and odd:
354 is not to be included in the Venn Diagram.
What are prime numbers ?Prime numbers are positive integers that have exactly two distinct divisors, which are 1 and the number itself. In other words, a prime number is a number greater than 1 that is divisible only by 1 and itself.
1 is an odd number but not a prime number. 2 is a prime number but not an odd number. 3 and 5 are both odd and prime numbers.
Find out more on prime numbers at https://brainly.com/question/881160
#SPJ1
The line on the graph passes through the points A (0, 6) and B (3, 0).
YA
a) Calculate the gradient of line AB.
b) Find the gradient of a line perpendicular
to AB.
c) Find the equation of the line passing
through point A and perpendicular
to AB.
B
The equation for the line passing through point A and perpendicular to AB will be y-0.5x=6, the gradient of line AB is -2, and the gradient of a line perpendicular to AB is 0.5.
What is the slope or gradient?A numerical assessment of a line's angle relative to the ground is known as the slope.
Given data;
m₁ is the slope of line AB
m₂ is the slope of a line perpendicular to AB
The coordinate points are,
A,(x₁,y₁)= (0, 6)
B,(x₂,y₂)=(3, 0).
The gradient of line AB;
\(\rm m_{{1} }= \frac{y_2-y_1}{x_2-x_1} \\\\ \rm m_{{1} }= \frac{0-6}{3-0} \\\\ m_{{1} }=-2\)
The slope of the lines has a perpendicular relation is -1;
m₁ × m₂ = -1
(-2) × m₂ = -1
m₂ = 1/2
m₂ = 0.5
The equation of the line passing through point A and perpendicular to AB;
(y - y₁) = m₂(x-x₁)
(y-6)=0.5(x-0)
y-6 = 0.5 x
y-0.5x=6
Hence, the gradient of line AB, the gradient of a line perpendicular to AB, and the equation of the line passing through point A and perpendicular to AB will be -2,0.5 and y-0.5x=6.
To learn more about the slope, refer to the link;
https://brainly.com/question/3605446
#SPJ1
6th grade math bro :)
Answer:
Answer 1 is 56
Answer 2 is 66
Step-by-step explanation:
1. 56/56 * 100 = 100%
2. 66 * 50/100 = 33
Friendship’s soccer team purchased uniforms and equipment for a total of $912. The equipment cost $612 and the uniforms cost $25 each. How many uniforms did the school purchase?
Answer:
12
Step-by-step explanation:
912 - 612 = 300
300 ÷ 25 = 12
The number of uniforms the school purchased based on the given data and equation formation will be $12.
How to form an equation?Determine the known quantities and designate the unknown quantity as a variable while trying to set up or construct a linear equation to fit a real-world application.
In other words, an equation is a set of variables that are constrained through a situation or case.
Let's suppose the school purchase x number of uniforms.
Total cost = equipment cost + x uniforms cost
912 = 612 + 25x
912 - 612 = 25x
25x = 300
x = 12
Hence "The number of uniforms the school purchased based on the given data and equation formation will be $12".
For more about the equation,
https://brainly.com/question/10413253
#SPJ2
Triangle ABC is transformed to Triangle A′B′C′, as shown below: A coordinate grid is shown from negative 4 to 0 to 4 on both x- and y-axes. A triangle ABC has A at ordered pair negative 3, 4, B at ordered pair negative 4, 2, C at ordered pair negative 1, 1. A triangle A prime B prime C prime has A prime at ordered pair 3, 4, B prime at ordered pair 4, 2, C prime at ordered pair 1, 1. Which equation shows the correct relationship between the measures of the angles of the two triangles? (5 points)
Answer:
Option 4.
\(\angle BCA =\angle B' C'A'\\\)
Step-by-step explanation:
By graphing these we see that these triangles are Similar which means that there corresponding angles measure is equal i-e all angles are equal and hence option 4 is the answer
Answer:
The measure of angle ABC = The measure of angle A prime B prime C prime
Step-by-step explanation:
Candice spent 3 ⅘ hours, and Shane spend 2 ⅒ hours at track practice over the weekend. How many more hours did Candice spend than Shane at track practice?
Answer:
1 7/10
Step-by-step explanation:
3 4/5 you have to times the fraction be to so it would be 3 8/10 so now you would take your 2 1/10 and subtract so it would be 1 7/10
Use the information provided below to calculate Samantha’s remuneration for 17 March 2022.
Samantha’s normal wage is R300 per hour and her normal working day is 8 hours. The standard production time for each employee is 4 units for every 30 minutes. On 17 March 2022, Samantha’s production was 76 units. Using the Halsey bonus system, a bonus of 50% of the time saved is given to employees.
Samantha’s remuneration for 17 March 2022 is R2475.
To calculate Samantha’s remuneration for 17 March 2022, we need to find the time saved by her using the Halsey bonus system.
Step-by-step explanation:
As per the given data,
Samantha’s normal wage is R300 per hour and her normal working day is 8 hours.
Thus, Samantha’s normal wage for a day = R300 × 8 = R2400.
The standard production time for each employee is 4 units for every 30 minutes.
Therefore, Production time for Samantha = 76 units
Time taken to produce 4 units = 30 minutes
Time taken to produce 76 units = (30/4) × 76 minutes
= 570 minutes (9.5 hours)
Therefore, Samantha took 9.5 hours to produce 76 units.
Using the Halsey bonus system, a bonus of 50% of the time saved is given to employees.
So, time saved = 9.5 − (76/4 × 0.5)
= 9.5 − 9
= 0.5 hours
= 30 minutes.
Remuneration for the day will be:
Samantha’s normal wage for a day + Bonus
Normal wage for 0.5 hour = 300 × 0.5
= R150
Bonus = 50% of time saved
= 50% of R150
= R75
Therefore, Remuneration for Samantha for 17 March 2022 = R2400 + R75
= R2475.
Answer: Samantha’s remuneration for 17 March 2022 is R2475.
To know more about remuneration visit:
https://brainly.com/question/31022519
#SPJ11
(6, 3), (7, - 4) a -7 b . 1 c . 7 d . - 1/7
Answer:
The answer is a. -7
Step-by-step explanation:
Given,
\(A=(6,3)\\B=(7,-4)\)
We know,
\(Coordinate\,\,plane,m=\frac{y_{2}-y_{1} }{x_{2}-x_{1}} \\So,Coordinate\,\,plane\,\,for\,\,AB,m=\frac{-4-3 }{7-6}=\frac{-7}{1}=-7\)
Hope you have understood this.....
ask me if you have any confusion.....
if you liked it pls mark it as the brainliest
how to calculate temperature of earth 3 billion years ago with albedo?
To calculate the temperature of the Earth 3 billion years ago with albedo, we need to first determine the albedo at that time. This can be done by analyzing various geological and chemical data, such as the composition of rocks and sediments, the presence of specific minerals and isotopes, and the amount of atmospheric gases and aerosols.
Once the albedo is estimated, we can use the Stefan-Boltzmann law, which relates the temperature of a body to its radiative energy emission, to determine the equilibrium temperature of the Earth. The Stefan-Boltzmann law states that the radiative energy emission of a body is proportional to the fourth power of its absolute temperature.
Assuming that the Earth was in radiative equilibrium with the Sun, we can use the following formula to calculate the temperature of the Earth:
T = (S (1 - A) / 4 σ)^(1/4)
where T is the temperature of the Earth, S is the solar constant (the amount of solar radiation received by the Earth), A is the albedo, and σ is the Stefan-Boltzmann constant.
By plugging in the values for S, A, and σ, we can calculate the temperature of the Earth at that time. However, it's important to note that there are many uncertainties and assumptions involved in this calculation, and the estimated temperature should be taken as an approximation.
The temperature of the Earth in the past can be estimated using various geological and climatological data. One important factor that can help in determining the temperature is the Earth's albedo, which is the proportion of solar radiation reflected by the Earth's surface and atmosphere. The albedo can affect the amount of energy the Earth receives from the Sun, which can impact the temperature.
Know more about "earth":-
https://brainly.com/question/13110278#
#SPJ11
Can someone help me understand this?
Answer:
the slope is 5. 5/1
Step-by-step explanation:
Use rise over run to find slope. It is the easiest way to find slope
You rise 5 and 1 to the right
Answer:
The slope is 5
Step-by-step explanation:
So you're just trying to find the slop. We're going to choose 2 points to use which are already marked on the graph.
(0, 2) and (-1, -3)
So to find the slope, we have to find (the difference in y) / (the difference in x)
So we're going to do
-3-2/-1-0
Which is
-5/-1
So since there are two negatives, it become positive, and it'll simplify into 5.
So the slope of the line is 5
statistical power is a measure of the ability to reject the null hypothesis when:
Statistical power is a measure of the ability to reject the null hypothesis when it is false. It represents the probability of correctly identifying a true effect or relationship in a statistical hypothesis test.
A high statistical power indicates a greater likelihood of detecting a significant result if the null hypothesis is indeed incorrect. The power of a statistical test depends on several factors, including the sample size, the effect size (the magnitude of the true effect or difference), the chosen significance level (often denoted as α), and the variability or noise in the data. Increasing the sample size or effect size generally increases the statistical power, while a lower significance level or higher variability decreases it.
Power analysis is commonly used to determine an appropriate sample size for a study, ensuring that it is adequately powered to detect the desired effect. A higher power is desirable as it reduces the chances of a Type II error (failing to reject the null hypothesis when it is false) and increases the chances of correctly detecting real effects or relationships.
Learn more about null hypothesis here:
https://brainly.com/question/29387900
#SPJ11
Using the line of best fit for the above data, what is the approximate difference between the actual value for x = 10 and the predicted value for x= 10?
A. 1.60
B. 1.65
C. 1.70
D. 1.75
Answer:
I dont know
Step-by-step explanation: